Những câu hỏi liên quan
Kiệt Nguyễn Văn
Xem chi tiết
Nguyễn Tất Đạt
2 tháng 9 2018 lúc 16:26

Ta đi c/m BĐT sau: \(x^3+y^3\ge xy\left(x+y\right)\) (*)

Thật vậy (*) \(\Leftrightarrow x^3+y^3-x^2y-xy^2\ge0\)

\(\Leftrightarrow x^2\left(x-y\right)+y^2\left(y-x\right)\ge0\)

\(\Leftrightarrow\left(x-y\right)\left(x^2-y^2\right)\ge0\)

\(\Leftrightarrow\left(x-y\right)^2\left(x+y\right)\ge0\)(luôn đúng)

Áp dụng vào bài toán: 

\(\frac{1}{x^3+y^3+1}\le\frac{1}{xy\left(x+y\right)+1}=\frac{1}{xy\left(x+y+z\right)}\)(Do xyz=1)

Tương tự: \(\frac{1}{y^3+z^3+1}\le\frac{1}{yz\left(x+y+z\right)};\frac{1}{z^3+x^3+1}\le\frac{1}{zx\left(x+y+z\right)}\)

\(\Rightarrow A\le\frac{1}{xy\left(x+y+z\right)}+\frac{1}{yz\left(x+y+z\right)}+\frac{1}{zx\left(x+y+z\right)}=\frac{x+y+z}{xyz\left(x+y+z\right)}=1\)

Vậy Max A = 1. Dấu "=" xảy ra <=> x=y=z=1.

Bình luận (0)
Nguyen Duy Dai
Xem chi tiết
Cố Tử Thần
14 tháng 8 2020 lúc 15:32

áp dụng bunhiacopski ta có: 

P^2 =< (1+1+1)(1/1+x^2 + 1/1+y^2+1/1+z^2)= 3(....)

đặt (...) =A

ta có: 1/1+x^2=< 1/2x

tt với 2 cái kia

=> A=< 1/2(1/x+1/y+1/z) =<1/2 ( xy+yz+xz / xyz)=1/2 ..........

đoạn sau chj chịu

^^ sorry

Bình luận (0)
 Khách vãng lai đã xóa
FL.Hermit
14 tháng 8 2020 lúc 15:48

Bài này là câu lớp 8 rất quen thuộc rùiiiiiii !!!!!!!!

gt <=>    \(\frac{x+y+z}{xyz}=1\)

<=>    \(\frac{1}{xy}+\frac{1}{yz}+\frac{1}{zx}=1\)

Đặt:   \(\frac{1}{x}=a;\frac{1}{y}=b;\frac{1}{z}=c\)

=>    \(ab+bc+ca=1\)

VÀ:    \(x=\frac{1}{a};y=\frac{1}{b};z=\frac{1}{c}\)

THAY VÀO P TA ĐƯỢC:    

\(P=\frac{1}{\sqrt{1+\frac{1}{a^2}}}+\frac{1}{\sqrt{1+\frac{1}{b^2}}}+\frac{1}{\sqrt{1+\frac{1}{c^2}}}\)

=>     \(P=\frac{1}{\sqrt{\frac{a^2+1}{a^2}}}+\frac{1}{\sqrt{\frac{b^2+1}{b^2}}}+\frac{1}{\sqrt{\frac{c^2+1}{c^2}}}\)

=>     \(P=\frac{a}{\sqrt{a^2+1}}+\frac{b}{\sqrt{b^2+1}}+\frac{c}{\sqrt{c^2+1}}\)

Thay     \(1=ab+bc+ca\)    vào P ta sẽ được:

=>      \(P=\frac{a}{\sqrt{a^2+ab+bc+ca}}+\frac{b}{\sqrt{b^2+ab+bc+ca}}+\frac{c}{\sqrt{c^2+ab+bc+ca}}\)

=>     \(P=\frac{a}{\sqrt{\left(a+b\right)\left(a+c\right)}}+\frac{b}{\sqrt{\left(b+a\right)\left(b+c\right)}}+\frac{c}{\sqrt{\left(c+a\right)\left(c+b\right)}}\)

=>      \(2P=2.\sqrt{\frac{a}{a+b}}.\sqrt{\frac{a}{a+c}}+2.\sqrt{\frac{b}{b+a}}.\sqrt{\frac{b}{b+c}}+2.\sqrt{\frac{c}{c+a}}.\sqrt{\frac{c}{c+b}}\)

TA ÁP DỤNG BĐT CAUCHY 2 SỐ SẼ ĐƯỢC:

=>      \(2P\le\frac{a}{a+b}+\frac{a}{a+c}+\frac{b}{b+a}+\frac{b}{b+c}+\frac{c}{c+a}+\frac{c}{c+b}\)

=>     \(2P\le\left(\frac{a}{a+b}+\frac{b}{b+a}\right)+\left(\frac{b}{b+c}+\frac{c}{c+b}\right)+\left(\frac{c}{c+a}+\frac{a}{a+c}\right)\)

=>     \(2P\le\frac{a+b}{a+b}+\frac{b+c}{b+c}+\frac{c+a}{c+a}\)

=>     \(2P\le1+1+1=3\)

=>     \(P\le\frac{3}{2}\)

DẤU "=" XẢY RA <=>    \(a=b=c\)    . MÀ     \(ab+bc+ca=1\)

=>     \(a=b=c=\sqrt{\frac{1}{3}}\)

=>     \(x=y=z=\sqrt{3}\)

VẬY P MAX \(=\frac{3}{2}\)      <=>      \(x=y=z=\sqrt{3}\)

Bình luận (0)
 Khách vãng lai đã xóa
Ngô quang minh
Xem chi tiết
alibaba nguyễn
9 tháng 11 2016 lúc 22:22

Đề bài mâu thuẫn quá. Cả x,y,z đều lớn hơn 0 thì làm sao xyz = 0 được

Bình luận (0)
Thắng Nguyễn
9 tháng 11 2016 lúc 22:55

Câu hỏi của Lâm Minh Anh - Toán lớp 9 - Học toán với OnlineMath

Bình luận (0)
Hexic Sami
21 tháng 10 2018 lúc 7:20

Bài này có người hỏi rồi

Bình luận (0)
hh hh
Xem chi tiết
nguyễn thùy linh
21 tháng 2 2017 lúc 21:26

x,y,z là số thực à khó đấy số dương thì mk còn làm đc 

chứ số thực mk chịu

Bình luận (0)
NGUYỄN THẾ HIỆP
21 tháng 2 2017 lúc 23:08

Biến đổi tương đương ta CM được BĐT sau: \(x^3+y^3\ge xy\left(x+y\right)\)

Ta có: \(\frac{1}{x^3+y^3+1}\le\frac{1}{xy\left(x+y\right)+xyz}=\frac{1}{xy\left(x+y+z\right)}=\frac{z}{xyz\left(x+y+z\right)}\)

CM tương tự với các phân thức còn lại

Cộng vế theo vế các BĐT đó ta được:

\(A\le\frac{x+y+z}{xyz\left(x+y+z\right)}=\frac{1}{xyz}=1\)

Vậy Max A=1 <=> x=y=z=1

Bình luận (0)
Nguyen Duy Dai
Xem chi tiết
nub
16 tháng 8 2020 lúc 19:52

Xét: \(x^4+y^4-xy\left(x^2+y^2\right)=\left(x^2+y^2+xy\right)\left(x-y\right)^2\ge0\)

\(\Rightarrow x^4+y^4\ge xy\left(x^2+y^2\right)\)(*)

Tương tự với (*) ta có: \(\hept{\begin{cases}y^4+z^4\ge yz\left(y^2+z^2\right)\\z^4+x^4\ge zx\left(z^2+x^2\right)\end{cases}}\)

\(\Rightarrow\Sigma_{cyc}\frac{1}{x^4+y^4+z}\le\Sigma_{cyc}\frac{1}{xy\left(x^2+y^2\right)+z.xyz}=\Sigma_{cyc}\frac{1}{xy\left(x^2+y^2+z^2\right)}=\frac{x+y+z}{x^2+y^2+z^2}\)

Ta có:\(x^2+y^2+z^2\ge\frac{1}{3}\left(x+y+z\right)^2\) và \(x+y+z\ge3\sqrt[3]{xyz}=3\)

\(\Rightarrow\Sigma_{cyc}\frac{1}{x^4+y^4+z}\le\frac{x+y+z}{x^2+y^2+z^2}\le\frac{1}{\frac{1}{3}\left(x+y+z\right)}\le1\)

Dấu "=" xảy ra khi x=y=z=1

Bình luận (0)
 Khách vãng lai đã xóa
NBH
Xem chi tiết
Hoàng Phúc
25 tháng 1 2017 lúc 20:23

đầu tiên cần c/m x3+y3 >= xy(x+y) (chứng minh=biến đổi tương đương)

 ta có x3+y3+1 >= xy(x+y)+1=xy(x+y)+xyz=xy(x+y+z)

=>1/(x3+y3+1) <= 1/xy(x+y+z)

tương tự với 2 phân thức còn lại rồi cộng lại

Bình luận (0)
phamthibinh
27 tháng 1 2017 lúc 0:54

Cộng lại chưa cái gì cả

Bình luận (0)
Nguyễn Thanh Hiền
Xem chi tiết
hinata shouyou
15 tháng 4 2019 lúc 20:43

x;y;z có dương không bạn 

Bình luận (0)
kudo shinichi
15 tháng 4 2019 lúc 20:46

Đặt\(\hept{\begin{cases}x=a^3\\y=b^3\\z=c^3\end{cases}}\)

Có: \(xyz=1\Rightarrow a^3b^3c^3=1\Leftrightarrow abc=1\)

\(\Rightarrow\frac{1}{1+x+y}=\frac{1}{a^3+b^3+abc}\)

Ta có: \(a^3+b^3\ge ab\left(a+b\right)\)( tự c/m)

dấu " = " xảy ra <=> a=b

\(\Rightarrow\frac{1}{1+x+y}=\frac{1}{a^3+b^3+abc}\le\frac{1}{ab\left(a+b\right)+abc}=\frac{1}{\left(a+b+c\right)ab}\)

Tương tự: \(\frac{1}{1+z+y}=\frac{1}{c^3+b^3+abc}\le\frac{1}{cb\left(c+b\right)+abc}=\frac{1}{\left(a+b+c\right)cb}\)

                \(\frac{1}{1+z+x}=\frac{1}{c^3+a^3+abc}\le\frac{1}{ca\left(c+b\right)+abc}=\frac{1}{\left(a+b+c\right)ca}\)

Cộng  vế với vế của 3 BĐT trên ta có:

\(P\le\frac{1}{ab\left(a+b+c\right)}+\frac{1}{bc\left(a+b+c\right)}+\frac{1}{ca\left(a+b+c\right)}=\frac{a+b+c}{abc\left(a+b+c\right)}=\frac{1}{abc}=1\)

Dấu " = " xảy ra <=> x=y=z=1

Vậy \(P_{max}=1\Leftrightarrow x=y=z=1\)

Bình luận (0)
hinata shouyou
15 tháng 4 2019 lúc 20:48

kudo shinichi , x,y,z phải dương ms làm được như bạn

Bình luận (0)
forever young
Xem chi tiết
Ngọc Nguyễn
Xem chi tiết
ღ๖ۣۜLinh
17 tháng 8 2019 lúc 12:21

Đặt \(^{\hept{\begin{cases}x=a^2\\y=b^2\\z=c^2\end{cases}}\Rightarrow abc=1}\)

\(\Rightarrow P=\frac{1}{a^2+2b^2+3}+\frac{1}{b^2+2c^2+3}+\frac{1}{c^2+2a^2+3}\)

ÁP DỤNG BĐT AM-GM : 

\(a^2+b^2\ge2ab\)

\(b^2+1\ge2b\)

\(\Rightarrow a^2+2b^2+3\ge2\left(ab+b+1\right)\)

\(\Rightarrow\frac{1}{a^2+2b^2+3}\le\frac{1}{2}.\frac{1}{ab+b+1}\)

Tương tự \(\frac{1}{b^2+2c^2+3}\le\frac{1}{2}.\frac{1}{bc+c+1}\)

               \(\frac{1}{c^2+2a^2+3}\le\frac{1}{2}.\frac{1}{ac+a+1}\)

Cộng từng vế các bđt trên ta được

\(P\le\frac{1}{2}\)

Dấu "=" xảy ra khi x=y=z=1

Bình luận (0)